LSAT and Law School Admissions Forum

Get expert LSAT preparation and law school admissions advice from PowerScore Test Preparation.

 Administrator
PowerScore Staff
  • PowerScore Staff
  • Posts: 8929
  • Joined: Feb 02, 2011
|
#33152
Complete Question Explanation

Justify the Conclusion—SN. The correct answer choice is (D)

This Justify the Conclusion question gives you a tremendous opportunity to develop a precision prephrase, because the conclusion contains new information, not provided in a premise. Because this information must be provided by the correct answer choice, your prephrase can be very precise.

The stimulus begins with a conditional relationship: if the budget does not allow for more dairy inspectors to be hired, most of the large dairies in the central valley will not meet federal standards governing the disposal of natural wastes.

You can diagram this relationship as:

BA = budget allows for more dairy inspectors to be hired
MS = most large dairies will meet federal standards governing the disposal of natural wastes
  • BA ..... :arrow: ..... MS
Not properly disposing of natural wastes is a problem, because natural wastes can seep into streams and groundwater. Next, you are told that the new district budget does not allow for the hiring of more dairy inspectors, meaning that the sufficient condition from the first sentence (i.e., BA) has been met.

You can infer, then, that the necessary condition must be fulfilled, that most large dairies will not meet federal standards governing the disposal of natural wastes (i.e., MS). However, this is not the conclusion reached by the stimulus author. Instead, the conclusion is that the district’s drinking water is likely to become polluted.

This information about the drinking water being likely to become polluted has appeared for the first time in the conclusion. You may feel the stimulus discussed this possibility when it told you that the natural waste can seep into streams and groundwater. Do not fall for this trap. Certainly, these ideas are connected. But they are not the same.

You are not permitted to assume that the district gets it drinking water from the streams and groundwater. And, even if that assumption were permissible, it would not necessarily be the case that most of the district’s drinking water is likely to become polluted. The inclusion of the word “most” in the conclusion is intended to remind you of the use of the word “most” in the first sentence. However, just because most of the large dairies in the central valley do not meet the federal standards does not justify the conclusion that most of the district’s drinking water is likely to become polluted.

Your very mechanical prephrase in this Justify the Conclusion question is that the correct answer choice will explicitly connect these previously unconnected ideas in a definitive way that proves the conclusion is valid, such that if most of the large dairies in the central valley do not meet the federal standards governing the disposal of natural wastes, then most of the district’s drinking water is likely to become polluted:
  • MS ..... :arrow: ..... most drinking water likely to be become polluted
Answer choice (A): This answer choice does not justify the conclusion, because it deals with the situation in which most of the dairies, and not more specifically most of the large dairies, meet the federal standard. In the stimulus, the evidence was that most of the large dairies will not meet the federal standard.

Answer choice (B): Information about what is required to keep the drinking water in the district clean does not justify the conclusion that most of the district’s drinking water is likely to become polluted.

Answer choice (C): This answer choice is incorrect, because it states only what is required for it to be likely that most of the district’s drinking water will become polluted, rather than information that would prove that most of the district’s drinking water is likely to become polluted.

Answer choice (D): This is the correct answer choice. This answer choice presents the prephrase described above. It is the nature of the Justify the Conclusion answer choice, which must prove the validity of the stimulus, that permitted such a precise prephrase for this question.

Answer choice (E): This answer choice does not prove the validity of the conclusion, because it relies on different, and stronger, evidence than what was provided. The stimulus merely provided that most of the large dairies in the central valley will fail to meet the federal standards, not that none of the large dairies in the central valley will fail to meet the federal standards.
 JSLSAT
  • Posts: 29
  • Joined: Jul 06, 2016
|
#28039
Hi PowerScore,

I'm having trouble disproving answers B & E.

I can see that our stimulus says:

/Budget Allows More Inspectors Hired --> /Most Large Dairies in Central Valley Meet Federal Disposal Standards
/Budget Allows More Inspectors Hired
---------------
Most of the district's water is likely to become polluted

And that the sufficient assumption needs to include something that combines disposal standards with district water pollution. Answer D does this perfectly.

But answer B appears to state:

All drinking water clean --> Hire more inspectors

The contrapositive would be:
/Budget Allows More Inspectors Hired --> Not all drinking water clean

Wouldn't that allow us to conclude that MOST of the district's drinking water polluted? Or does it limit us to just "some"?

Answer E appears to state:

/Large Dairies in the Central Valley meet federal standards of disposal --> Most drinking water becomes polluted

Doesn't "All" encapsulate "Most"? So wouldn't the above be a match for our stimulus conditions, that if ALL large dairies fail to meet the standards then MOST fail to meet those standards as well?
 Emily Haney-Caron
PowerScore Staff
  • PowerScore Staff
  • Posts: 577
  • Joined: Jan 12, 2012
|
#28049
Hi JSLSAT,

I'm going to address each part of your question separately.
But answer B appears to state:

All drinking water clean --> Hire more inspectors

The contrapositive would be:
/Budget Allows More Inspectors Hired --> Not all drinking water clean

Wouldn't that allow us to conclude that MOST of the district's drinking water polluted? Or does it limit us to just "some"?
For this portion of your question, you figured it out on your own. The problem here is one of degree; B would just allow us to conclude that some of the drinking water would be polluted.
Answer E appears to state:

/Large Dairies in the Central Valley meet federal standards of disposal --> Most drinking water becomes polluted

Doesn't "All" encapsulate "Most"? So wouldn't the above be a match for our stimulus conditions, that if ALL large dairies fail to meet the standards then MOST fail to meet those standards as well?
The problem with E isn't "all" vs. "most," it is "none" vs. "most." The stimulus is saying most of the large dairies will not meet federal standards, implying some will. But E is only triggered if none of them meet federal standards.

I hope that helps!
 JSLSAT
  • Posts: 29
  • Joined: Jul 06, 2016
|
#28056
Thank you! So it's about focusing on the particular conclusion at hand - if it's "most", you can't accommodate that with an "all" or a "some", it needs to be an additional "must" condition?

Again, many thanks!
 Nikki Siclunov
PowerScore Staff
  • PowerScore Staff
  • Posts: 1362
  • Joined: Aug 02, 2011
|
#28073
Hi JSLSAT,

Let me add my 2c here.

The key to an optimal approach here is to differentiate key operands such as "some," "most," and "all." Whereas "all" clearly entails "most," and "most" entails "some," the reverse is not necessarily true. All that answer choice (B) allows us to conclude is that not all of the drinking water is going to be clean. In other words, some of it will be polluted. Since "some" does not prove "most," answer choice (B) is not strong enough to justify the conclusion (which is that most of the drinking water is likely to become polluted).

I wouldn't go as far as to suggest a keyword-matching strategy here, but it is imperative to know what you're trying to prove--and what answer choices are likely to fall short.

Thanks,
 JSLSAT
  • Posts: 29
  • Joined: Jul 06, 2016
|
#28108
I believe I understand what you're getting at - in this case, "most" triggers the result, whereas "all" may trigger a different result. Instead of matching "most", it's knowing which actually triggers the necessary condition we're seeking. A rough example might be, "If MOST of the cookies are gone, I will buy a new box next week". But "If ALL of the cookies are gone" may have a very different necessary condition that we're unaware of.
 Emily Haney-Caron
PowerScore Staff
  • PowerScore Staff
  • Posts: 577
  • Joined: Jan 12, 2012
|
#28175
Hi JSLSAT,

When thinking about Nikki's response, it might help to really think through the description provided of the sort of directional nature of these terms; if you know you have "most," that means you will definitely have "some," but if you know you have "some," that doesn't tell you whether you also have "most." So to use the example you gave, if I tell you that "If ALL of the cookies are gone, I will buy a new box next week," and then you find out that MOST of the cookies are gone, you don't have any idea if I'm going to buy a new box next week. Does that make sense?
 JSLSAT
  • Posts: 29
  • Joined: Jul 06, 2016
|
#28724
I think I see what you mean - a MOST condition can be triggered by an ALL, but an ALL condition cannot be triggered by a most. Is that accurate? Many thanks!
User avatar
 Dave Killoran
PowerScore Staff
  • PowerScore Staff
  • Posts: 5862
  • Joined: Mar 25, 2011
|
#28730
JSLSAT wrote:I think I see what you mean - a MOST condition can be triggered by an ALL, but an ALL condition cannot be triggered by a most. Is that accurate? Many thanks!

Yes, that is accurate. Good job!
 15veries
  • Posts: 113
  • Joined: Sep 25, 2016
|
#29598
I was also not sure between D and E.
I thought since this is sufficient assumption, bigger picture is better...so chose E.
If none is true, most is true too...
is this an example of a question that not all sufficient assumption go beyond its scope?
Plus I thought D's "most" and stimulus's "most" may not overlap, so I thought D is wrong...

Thanks

Get the most out of your LSAT Prep Plus subscription.

Analyze and track your performance with our Testing and Analytics Package.